Discrete Mathematics with Applications 4th Edition

Published by Cengage Learning
ISBN 10: 0-49539-132-8
ISBN 13: 978-0-49539-132-6

Chapter 3 - The Logic of Quantified Statements - Exercise Set 3.3 - Page 130: 29

Answer

a. Version with interchanged quantifiers: ∃x ∈ R such that ∀y ∈ R, x < y. b. The given statement is true. The statement with exchanged quantifiers (part a) is false.

Work Step by Step

b. The given statement says that for any real number x, there is a real number y that is greater than x. This is true: For any real number x, let y = x + 1. Then x < y. The version with interchanged quantifiers says that there is a real number that is less than every other real number. This is false.
Update this answer!

You can help us out by revising, improving and updating this answer.

Update this answer

After you claim an answer you’ll have 24 hours to send in a draft. An editor will review the submission and either publish your submission or provide feedback.